LSAT and Law School Admissions Forum

Get expert LSAT preparation and law school admissions advice from PowerScore Test Preparation.

 Administrator
PowerScore Staff
  • PowerScore Staff
  • Posts: 8916
  • Joined: Feb 02, 2011
|
#34807
Complete Question Explanation

Main Point. The correct answer choice is (A)

For the second question in a row, the conclusion is more difficult than normal to find because the
stimulus author does not use a conclusion indicator. Here, the stimulus begins with the statement that
if the temperature of electric stovetop burners was capped at 350° C, then the burners would cause
fewer fires. The reason given in support of this statement, which is the argument’s conclusion, is that
cooking oil and most common fibers do not ignite at less than 387° C. This is a simply structured
one-premise, one-conclusion argument.

In addition to the premise and conclusion, the argument contains two other pieces of information: a
temperature of 350° C provides enough heat for efficient and effective cooking; and electric burners
can reach temperatures well above 700° C. These facts provide context, but are neither premises nor
conclusions of the argument. Respectively, they tell us that the suggested change is practical and
necessary.

This is a Main Point question. Our prephrase is that the correct answer choice will restate the
argument’s conclusion, that “electric stovetop burners would cause fewer fires if their highest
temperature was limited to 350° C (662° F).

Answer choice (A): This is the correct answer choice, and is a nearly perfect restatement of the
conclusion. All that is missing is the parenthetical statement of the proposed temperature limit in
Fahrenheit.

Answer choice (B): This choice restates one of the context-providing statements from the stimulus,
and does not restate the conclusion.

Answer choice (C): This answer choice describes a premise of the argument.

Answer choice (D): As with answer choice (A), this choice restates a fact offered in the stimulus to
provide context to the argument, rather than restating the conclusion.

Answer choice (E): While this statement of causality was implied by the argument, it was not
explicitly stated anywhere.

Get the most out of your LSAT Prep Plus subscription.

Analyze and track your performance with our Testing and Analytics Package.